Cardiovascular

Réussis tes devoirs et examens dès maintenant avec Quizwiz!

MI Complications 1. Thrombus > embolism CVA/stroke 2. Decreased contractility > shock/CHF, 3. Tissue necrosis > expanding infarct, papillary mm infarct (MR murmur), VSD, ventricular rupture/ pericarditis (cardiac tamponade) 4. Electrical instability (arrhythmias, most common cause of few day post MI death = VT and VF)

1. Pump failure (CHF) a. Most common cause of in-hospital mortality b. If mild, treat medically (ACE inhibitor, diuretic) c. If severe, may lead to cardiogenic shock; invasive hemodynamic monitoring may be indicated 2. Arrhythmias a. Premature ventricular contractions (PVCs)—conservative treatment (observation) indicated; no need for antiarrhythmic agents b. Atrial fibrillation c. Ventricular tachycardia (VT)-sustained VT requires treatment: If patient is hemodynamically unstable, electrical cardioversion is indicated. If patient is hemodynamically stable, start antiarrhythmic therapy (IV amiodarone)—see treatment of VT. d. VFib—immediate unsynchronized defibrillation and CPR are indicated (see Arrhythmias on page 22) e. Accelerated idioventricular rhythm-does not affect prognosis; no treatment needed in most cases. f. Paroxysmal supraventricular tachycardia (PSVT)—for treatment, see Arrhythmias. g. Sinus tachycardia • May be caused by pain, anxiety, fever, pericarditis, medications, etc. • Worsens ischemia (increases myocardial oxygen consumption) • Treat underlying cause (analgesics for pain, aspirin for fever, etc.) h. Sinus bradycardia • A common occurrence in early stages of acute MI • May be a protective mechanism (reduces myocardial oxygen demand) • No treatment is required other than observation. If bradycardia is severe or symptomatic (hemodynamic compromise), atropine may be helpful in increasing HR. i. Asystole • Very high mortality • Treatment should begin with electrical defibrillation for VFib, which is more common in cardiac arrest and may be difficult to clearly differentiate from asystole. • If asystole is clearly the cause of arrest, transcutaneous pacing is the appropriate treatment. j. AV block • Associated with ischemia involving conduction tracts • First-degree and second-degree (type I) blocks do not require therapy. • Second-degree (type II) and third-degree blocks: Prognosis is dire in the setting of an anterior MI—emergent placement of a temporary pacemaker is indicated (with later placement of a permanent pacemaker). In inferior MI, prognosis is better, and IV atropine may be used initially. If conduction is not restored, a temporary pacemaker is appropriate. 3. Recurrent infarction (extension of existing infarction or reinfarction of a new area) a. Both short-term and long-term mortality are increased. b. Diagnosis is often difficult. • Cardiac enzymes are already elevated from the initial infarction. Troponin levels remain elevated for a week or more, so are not useful here. CK-MB returns to normal faster, and so a re-elevation of CK-MB after 36 to 48 hours may be due to recurrent infarction. • If there is repeat ST segment elevation on ECG within the first 24 hours after infarction, suspect recurrent infarction. c. Treatment: Repeat thrombolysis or urgent cardiac catheterization and PCI. Continue standard medical therapy for MI. 4. Mechanical complications a. Free wall rupture • A catastrophic, usually fatal event that occurs during the first 2 weeks after MI (90% within 2 weeks, most commonly 1 to 4 days after MI) • 90% mortality rate • Usually leads to hemopericardium and cardiac tamponade • Treatment: Hemodynamic stabilization, immediate pericardiocentesis, and surgical repair b. Rupture of interventricular septum • Greater potential for successful therapy than with a free wall rupture, although this is also a critical event; emergent surgery is indicated • Occurs within 10 days after MI • Likelihood of survival correlates with size of defect c. Papillary muscle rupture • Produces mitral regurgitation • If suspected, obtain an echocardiogram immediately • Emergent surgery is needed (mitral valve replacement is usually necessary), as well as afterload reduction with sodium nitroprusside or intraaortic balloon pump (IABP). d. Ventricular pseudoaneurysm • Incomplete free wall rupture (myocardial rupture is contained by pericardium) • Bedside echocardiogram may show the pseudoaneurysm • Surgical emergency because ventricular pseudoaneurysms tend to become a free wall rupture e. Ventricular aneurysm • Rarely rupture (in contrast to pseudoaneurysms) • Associated with a high incidence of ventricular tachyarrhythmias • Medical management may be protective. • Surgery to remove aneurysm may be appropriate in selected patients. 5. Acute pericarditis a. The incidence has decreased sharply since the introduction of revascularization techniques. b. Treatment consists of aspirin (which is already standard in treatment of MI). c. NSAIDs and corticosteroids are contraindicated (may hinder myocardial scar formation). 6. Dressler's syndrome ("postmyocardial infarction syndrome") a. Immunologically based syndrome consisting of fever, malaise, pericarditis, leukocytosis, and pleuritis, occurring weeks to months after an MI b. Aspirin is the most effective therapy. Ibuprofen is a second option.

3rd degree AV block

Dx: 1. Absence of conduction of atrial impulses to the ventricles; no correspondence between P waves and QRS complexes 2. A ventricular pacemaker (escape rhythm) maintains a ventricular rate of 25 to 40 bpm. Characterized by AV dissociation (no AV relation at all) Tx: 1. Pacemaker implantation is necessary.

Mycotic aneurysm

Dx: 1. An aneurysm resulting from damage to the aortic wall secondary to infection 2. Blood cultures are positive in most cases. Tx: 1. IV antibiotics 2. Surgical excision

Peripheral vascular disease Intermittent claudication, rest pain, decreased pulses, ischemic ulcers

Dx: 1. Ankle-to-brachial index (ABI): Ratio of the systolic BP at the ankle to the systolic BP at the arm a. Normal ABI is between 0.9 and 1.3. b. ABI > 1.3 is due to noncompressible vessels and indicates severe disease (calcified stiff arteries). c. Claudication ABI < 0.7 d. Rest pain ABI < 0.4 2. Pulse volume recordings a. Excellent assessment of segmental limb perfusion b. Pulse wave forms represent the volume of blood per heart beat at sequential sites down the leg. c. A large wave form indicates good collateral flow. d. Noninvasive using pressure cuffs 3. Arteriography (contrast in vessels and radiographs) a. Gold standard for diagnosing and locating PVD Tx: 1. Conservative management for intermittent claudication a. Stop smoking (the importance of this cannot be overemphasized). Smoking is linked to progression of atherosclerosis and causes vasoconstriction (further decreasing blood flow). b. Graduated exercise program: Walk to point of claudication, rest, and then continue walking for another cycle c. Foot care (especially important in diabetic patients) d. Atherosclerotic risk factor reduction (control of hyperlipidemia, HTN, weight, diabetes, and so on) e. Avoid extreme temperature (especially extreme cold). f. Aspirin along with ticlopidine/clopidogrel have shown slight improvements in symptom relief. They are often used in these patients for stroke/MI prevention. g. Cilostazol is a PDE inhibitor which acts both by suppressing platelet aggregation and by directly dilating arterioles. 2. Surgical treatment a. Indications: Rest pain, ischemic ulcerations (tissue necrosis), severe symptoms refractory to conservative treatment that affects quality of life or work b. Options • Angioplasty—balloon dilation with or without stenting. Given the minimal risks and good chance of symptomatic relief for patients, this may be performed one or more times before a bypass is done. • Surgical bypass grafting—has a 5-year patency rate of 70% (immediate success rate is 80% to 90%).

Tricuspid regurgitation Clinical features 1. Usually asymptomatic unless the patient develops symptoms of RHF/pulmonary HTN. 2. Signs and symptoms of RVF (ascites, hepatomegaly, edema, JVD) 3. Pulsatile liver 4. Prominent V waves in jugular venous pulse with rapid y descent 5. Inspiratory S3 along LLSB may be present 6. Blowing holosystolic murmur a. At LLSB b. Intensified with inspiration; reduced during expiration or the Valsalva maneuver 7. Right ventricular pulsation along LLSB 8. AFib is usually present.

Dx: 1. Echocardiogram a. Quantifies amount of TR b. Identifies prolapse/flail of tricuspid valve leaflets c. Measures pulmonary pressures d. ECG: RV and RA enlargement Tx: 1. Treat any underlying etiology of symptomatic TR. 2. Diuretics for volume overload and venous congestion/edema 3. Treat left-sided heart failure, endocarditis, or pulmonary HTN. 4. Severe regurgitation may be surgically corrected if pulmonary HTN is not present. a. Native valve repair surgery b. Valvuloplasty of tricuspid ring c. Valve replacement surgery: Rarely performed

Acute arterial occlusion 1. Pain—acute onset. The patient can tell you precisely when and where it happened. The pain is very severe, and the patient may have to sit down or may fall to the ground. 2. Pallor 3. Polar (cold) 4. Paralysis 5. Paresthesias 6. Pulselessness (use Doppler to assess pulses)

Dx: 1. Arteriogram to define site of occlusion 2. ECG to look for MI, AFib 3. Echocardiogram for evaluation of valves, clot, MI Tx: 1. Main goal: Assess viability of tissues to salvage limb 2. Skeletal muscle can tolerate 6 hours of ischemia; perfusion should be reestablished within this time frame. 3. Immediately anticoagulate with IV heparin. 4. Emergent surgical embolectomy is indicated via cutdown and Fogarty balloon (The Fogarty balloon catheter is used for embolectomy—the catheter is inserted, the balloon is inflated, and the catheter is pulled out—the balloon brings the embolus with it.) 5. Bypass is reserved for embolectomy failure. 6. Thrombolytics can also be infused intraarterially with recombinant urokinase 7. Treat any complications such as compartment syndrome that may occur.

Atrial myxoma (sporadic or AD) Benign gelatinous growth, usually pedunculated and usually arising from the interatrial septum of the heart in the region of the fossa ovalis. It is the most common primary cardiac neoplasm.

Dx: 1. Atrial myxomas can embolize, leading to metastatic disease, or can cause relative valvular dysfunction. 2. Fatigue, fever, syncope, palpitations, malaise, and a low-pitched diastolic murmur that changes character with changing body positions (diastolic plop). Tx: 1. Surgical excision

Deep venous thrombosis Cause: Virchow's triad (endothelial injury, venous stasis, hypercoagulability) gives rise to venous thrombosis. Classic findings (all have very low sensitivity/specificity): a. Lower-extremity pain and swelling (worse with dependency/walking, better with elevation/rest) b. Homans' sign (calf pain on ankle dorsiflexion) c. Palpable cord d. Fever Low-molecular-weight heparin (lovenox) • Has longer half life than unfractionated heparin and can be dosed once daily • Can be given on outpatient basis • No need to follow aPTT levels • Is much more expensive than unfractionated heparin

Dx: 1. Available studies a. Doppler analysis and Duplex ultrasound • Initial test for DVT; noninvasive, but highly operator dependent • High sensitivity and specificity for detecting proximal thrombi (popliteal and femoral), not so for distal (calf vein) thrombi b. Venography • Most accurate test for diagnosis of DVT of calf veins • Invasive and infrequently used • Allows visualization of the deep and superficial venous systems, and allows assessment of patency and valvular competence c. Impedance plethysmography • A noninvasive alternative to Doppler ultrasound • Blood conducts electricity better than soft tissue, so electrical impedance decreases as blood volume increases. • High sensitivity for proximal DVT, but not for distal DVT (calf veins) • Poor specificity because there is a high rate of false positives • As accurate as Doppler, but less operator dependent d. d-dimer testing • Has a very high sensitivity (95%), but low specificity (50%); can be used to rule out DVT when combined with Doppler and clinical suspicion 2. Interpretation of diagnostic tests a. Intermediate-to-high pretest probability of DVT • If Doppler ultrasound is positive, begin lovenox anticoagulation. • If Doppler ultrasound is nondiagnostic, repeat ultrasound every 2 to 3 days for up to 2 weeks. b. Low to intermediate probability of DVT • If Doppler ultrasound is negative, there is no need for anticoagulation; observation is sufficient. • Repeat ultrasound in 2 days. Tx: 1. Anticoagulation a. Prevents further propagation of the thrombus b. Heparin bolus followed by a constant infusion and titrated to maintain the PTT at 1.5 to 2 times aPTT c. Start warfarin once the aPTT is therapeutic and continue for 3 to 6 months. Keep INR at 2.0 to 3.0. d. Continue heparin until the INR has been therapeutic for 48 hours. 2. Thrombolytic therapy (streptokinase, urokinase, tissue plasminogen activator [t-PA]) a. Speeds up the resolution of clots b. Indicated for patients with massive PE, patients who are hemodynamically unstable, those with evidence of right heart failure (thrombolysis can reverse this), and those with no contraindications for thrombolytics. 3. Prophylactic inferior vena cava filter placement (Greenfield filter) a. For patients at high risk who have an absolute contraindication to other forms of prophylaxis b. Effective only in preventing PE, not DVT 4. Methods of prophylaxis after surgery a. Mechanical • Leg elevation, graduated compression stockings, early ambulation • Pneumatic compression boots—intermittently inflate and deflate, causing compression of the limb, usually the calves; very effective b. Pharmacologic • Heparin or LMWH: Unfractionated heparin or LMWH postoperatively until patient is ambulatory. • Combination of pneumatic compression devices and pharmacologic prophylaxis may provide the greatest protection.

Patent ductus arteriosus Signs Loud P2 (sign of pulmonary HTN) Continuous "machinery murmur" at left 2nd ICS (both systolic and diastolic components)

Dx: 1. CXR a. Increased pulmonary vascular markings b. Dilated pulmonary artery c. Enlarged cardiac silhouette d. Sometimes calcifications of ductus arteriosus 2. Echocardiography reveals the patent ductus and/or turbulent blood flow. Tx: 1. If pulmonary vascular disease is absent: Surgical ligation 2. If severe pulmonary HTN or right-to-left shunt is present, do not correct PDA. Surgery is contraindicated. 3. Indomethacin indicated for closure. Prostaglandin E1 can be used to keep the PDA open (may be needed in the face of other cardiac abnormalities such as transposition of the great vessels.)

Aortic dissection a. Type A (proximal) involves the ascending aorta (includes retrograde extension from descending aorta). b. Type B (distal) is limited to the descending aorta (distal to the take-off of the subclavian artery). The diagnosis of aortic dissection is very difficult to make because the classic clinical findings often are not apparent. The use of thrombolytic therapy in patients with aortic dissection who have been incorrectly diagnosed as having an acute MI can have fatal consequences.

Dx: 1. CXR shows widened mediastinum (>8 mm AP view). 2. TEE has a very high sensitivity and specificity; it is noninvasive and can be performed on unstable patients at the bedside. 3. CT scan and MRI are both highly accurate (MRI more so); MRI takes longer to perform, making it less ideal in the acute setting. 4. Aortic angiography is invasive, but it is the best test for determining the extent of the dissection for surgery. Tx: 11. Initiate medical therapy immediately. a. IV β-blockers to lower heart rate and diminish the force of left ventricular ejection b. IV sodium nitroprusside to lower systolic BP below 120 mm Hg 2. For type A dissections—surgical management a. Most cases of type A dissections should be treated as surgical emergencies to prevent complications such as MI, aortic regurgitation, or cardiac tamponade. b. Open surgery is still the standard of care. 3. For type B dissections—medical management a. Lower blood pressure as quickly as possible. First-line drugs include IV betablockers such as labetalol, esmolol, or propranolol. b. Pain control with morphine or dilaudid c. Unrelenting symptoms may require surgical treatment—both open and endovascular surgical options exist.

Aortic stenosis Signs a. Murmur • Harsh crescendo-decrescendo systolic murmur • Heard in second right intercostal space • Radiates to carotid arteries b. Soft S2. S2 may also be single since the aortic component may be delayed and merge into P2. c. S4 d. Parvus et tardus—diminished and delayed carotid upstrokes e. Sustained PMI f. Precordial thrill

Dx: 1. CXR: Calcific aortic valve, enlarged LV/LA (late) 2. ECG: LVH, LA abnormality a. Echocardiogram—diagnostic in most cases. Findings include LVH; thickened, immobile aortic valve; and dilated aortic root. 3. Cardiac catheterization a. Definitive diagnostic test b. Can measure valve gradient and calculate valve area (<0.8 cm2 indicates severe stenosis); normal aortic valve is 3 to 4 cm2 c. Useful in symptomatic patients before surgery Tx: Management of aortic stenosis is straightforward: • If asymptomatic: No treatment • If symptomatic: Surgery (aortic valve replacement) 1. Medical therapy has a limited role. 2. Surgical therapy: Aortic valve replacement is the treatment of choice. It is indicated in all symptomatic patients. 3. Percutaneous balloon valvuloplasty for AS has produced poor results (restenosis is a common problem).

Mitral regurgitation Signs a. Holosystolic murmur (starts with S1 and continues on through S2) at the apex, which radiates to the back or clavicular area, depending on which leaflet is involved b. AFib is a common finding c. Other findings: Diminished S1, widening of S2, S3 gallop; laterally displaced PMI; loud, palpable P2

Dx: 1. CXR: Cardiomegaly, dilated LV, pulmonary edema 2. Echocardiogram: MR; dilated LA and LV; decreased LV function Tx: 1. Medical a. Afterload reduction with vasodilators is recommended for symptomatic patients only; they are not recommended in most asymptomatic patients as they may mask progression of the disease b. Chronic anticoagulation if patient has atrial fibrillation c. IABP as bridge to surgery for acute MR 2. Surgical a. Mitral valve repair or replacement b. Must be performed before left ventricular function is too severely compromised

Aortic regurgitation Physical examination a. Widened pulse pressure—markedly increased systolic BP, with decreased diastolic BP b. Diastolic decrescendo murmur best heard at left sternal border c. Corrigan's pulse (water-hammer pulse)—rapidly increasing pulse that collapses suddenly as arterial pressure decreases rapidly in late systole and diastole; can be palpated at wrist or femoral arteries d. Austin-Flint murmur—low-pitched diastolic rumble due to competing flow anterograde from the LA and retrograde from the aorta. It is similar to the murmur appreciated in mitral stenosis. e. Displaced PMI (down and to the left) and S3 may also be present. f. Murmur intensity increases with sustained handgrip. Handgrip increases SVR, which causes an increased "backflow" through the incompetent aortic valve. g. De Musset's sign: Head bobbing h. Müller's sign: Uvula bobs i. Duroziez's sign: Pistol-shot sound heard over the femoral arteries

Dx: 1. CXR: LVH, dilated aorta 2. ECG: LVH 3. Echocardiogram—perform serially in chronic, stable patients to assess need for surgery a. Assess LV size and function. b. Look for dilated aortic root and reversal of blood flow in aorta. c. In acute aortic regurgitation, look for early closure of mitral valve. 4. Cardiac catheterization: To assess severity of aortic regurgitation and degree of LV dysfunction Tx: 1. Conservative if stable and asymptomatic: Salt restriction, diuretics, vasodilators, digoxin, afterload reduction (i.e., ACE inhibitors or arterial dilators), and restriction on strenuous activity 2. Definitive treatment is surgery (aortic valve replacement). This should be considered in symptomatic patients, or in those with significant LV dysfunction on echocardiogram. 3. Acute AR (e.g., post-MI): Medical emergency—perform emergent aortic valve replacement! 4. Endocarditis prophylaxis before dental and GI/genitourinary procedures

Mitral stenosis Signs a. Mitral stenosis murmur • The opening snap is followed by a low-pitched diastolic rumble and presystolic accentuation. This murmur increases in length as the disease worsens. • Heard best with bell of stethoscope in left lateral decubitus position b. S2 is followed by an opening snap. The distance between S2 and the opening snap can give an indication as to the severity of the stenosis. The closer the opening snap follows S2, the worse is the stenosis. c. Murmur is followed by a loud S1. A loud S1 may be the most prominent physical finding. d. With long-standing disease, will find signs of RVF (e.g., right ventricular heave, JVD, hepatomegaly, ascites) and/or pulmonary HTN (loud P2) e. All signs and symptoms will increase with exercise and during pregnancy.

Dx: 1. CXR: Left atrial enlargement (early) 2. Echocardiogram—most important test in confirming a. Left atrial enlargement b. Thick, calcified mitral valve c. Narrow, "fish mouth"-shaped orifice d. Signs of RVF if advanced disease Tx: 1. Medical a. Diuretics—for pulmonary congestion and edema b. Beta-blockers—to decrease heart rate and cardiac output c. Infective endocarditis prophylaxis d. Chronic anticoagulation with warfarin is indicated (especially if patient has AFib) 2. Surgical (for severe disease) a. Percutaneous balloon valvuloplasty usually produces excellent results. b. Open commissurotomy and mitral valve replacement are other options if valvotomy is contraindicated. 3. Management a. No therapy is required in asymptomatic patients. b. Diuretics can be used if the patient has mild symptoms. c. If symptoms are more severe, surgical treatment is recommended. d. If AFib develops at any time, treat accordingly (see discussion on AFib).

Leutic heart

Dx: 1. Complication of syphilitic aortitis, usually affecting men in their fourth to fifth decade of life. 2. Aneurysm of the aortic arch with retrograde extension backward to cause aortic regurgitation and stenosis of aortic branches, most commonly the coronary arteries. Tx: 1. IV penicillin 2. Surgical repair

Acute decompensated heart failure

Dx: 1. Diagnostic tests include ECG, chest x-ray, ABG, B-type natriuretic peptide (BNP), echocardiogram, possible coronary angiogram if indicated. 2. Hospital admission is indicated. Daily assessment of patient weight is a good method of documenting effective diuresis Tx: 1. Oxygenation and ventilatory assistance with non-rebreather face mask, NPPV, or even intubation as indicated. 2. Diuretics to treat volume overload and congestive symptoms—this is the most important intervention. Decreases preload. 3. Dietary sodium restriction. 4. Nitrates—IV nitroglycerin (vasodilator) in patients without hypotension. Decreases afterload. 5. Patients who have pulmonary edema despite use of oxygen, diuretics, and nitrates may benefit from use of inotropic agents (dobutamine). Digoxin takes several weeks to work and is not indicated in an acute setting. 6. There is limited evidence for use of morphine sulfate.

Infective endocarditis Always suspect endocarditis in a patient with a new heart murmur and unexplained fever. Acute endocarditis • Most commonly caused by S. aureus (virulent) • Occurs on a normal heart valve • If untreated, fatal in less than 6 weeks Subacute endocarditis • Caused by less virulent organisms, such as Streptococcus viridans and Enterococcus • Occurs on damaged heart valves • If untreated, takes much longer than 6 weeks to cause death

Dx: 1. Duke's clinical criteria: Two major criteria, one major and three minor criteria, or five minor criteria are required to diagnose infective endocarditis. 2. Major: • Sustained bacteremia by an organism known to cause endocarditis • Endocardial involvement documented by either echocardiogram (vegetation, abscess, valve perforation, prosthetic dehiscence) or clearly established new valvular regurgitation 3. Minor: • Predisposing condition (abnormal valve or abnormal risk of bacteremia) • Fever • Vascular phenomena: septic arterial or pulmonary emboli, mycotic aneurysms, intracranial hemorrhage, Janeway lesions • Immune phenomena: Glomerulonephritis, Osler's nodes, Roth's spots, rheumatoid factor • Positive blood cultures not meeting major criteria • Positive echocardiogram not meeting major criteria 4. TEE is better than transthoracic echocardiography in the diagnosis of endocarditis. Tx: 1. Parenteral antibiotics based on culture results for extended periods (4 to 6 weeks) 2. If cultures are negative but there is high clinical suspicion, treat empirically with a penicillin (or vancomycin) plus an aminoglycoside until the organism can be isolated. 3. Endocarditis prophylaxis (amoxicillin) is indicated for patients with known valvular heart disease or prosthetic valves who are about to undergo oral surgery or GI/GU surgery.

Myocardial infarction Cardiac Monitoring for a Patient With an Acute MI • BP and HR: HTN increases afterload and thus oxygen demand, whereas hypotension reduces coronary and tissue perfusion. Both nitrates and morphine can cause hypotension. • Rhythm strip with continuous cardiac monitor: Watch for dysrhythmias. Note that PVCs can lower stroke volume and coronary artery filling time. A high frequency of PVCs may predict VFib or VT. • Auscultate the heart (third and fourth heart sounds, friction rub, and so on) and lungs (crackles may indicate LV failure, pulmonary edema). • Hemodynamic monitoring (CVP, PCWP, SVR, cardiac index [CI]) with a pulmonary artery catheter is indicated if the patient is hemodynamically unstable. Monitoring is helpful in assessing the need for IV fluids and/or vasopressors. In MI, aspirin, β -blockers, and ACE inhibitors are the only agents shown to reduce mortality. (carvedilol reduces death risk with post MI LV dysfunction) Aspirin, β -blockers, and nitrates are used for stable angina, unstable angina, STEMI, and NSTEMI. The following are indicated in patients with MI: -Morphine -Oxygen -Nitroglycerin -Aspirin -Beta blocker -Statin Adjuvant MI: -ACEI -IV LMWH -Clopidogrel After an MI, all patients should be discharged home with aspirin, beta-blocker, statins, and an ACE inhibitor. Methods of Revascularization Percutaneous Coronary Intervention • This is the preferred treatment for STEMI as long as it can be performed expeditiously (door to balloon time less than 90 minutes) and by skilled personnel. • Also preferred in patients with contraindications for thrombolytic therapy; no risk of intracranial hemorrhage • PAMI trial showed that PTCA reduces mortality more than t-PA. Thrombolytic Therapy • Thrombolytic therapy remains an important treatment modality since PCI is still available only at specialized centers. It is useful for patients who present later, and for those in whom PCI is contraindicated. • Early treatment is crucial to salvage as much of the myocardium as possible. Administer as soon as possible up to 24 hours after the onset of chest pain. Outcome is best if given within the first 6 hours. • Indications: ST segment elevation in two contiguous ECG leads in patients with pain onset within 6 hours who have been refractory to nitroglycerin • Alteplase has been shown to have the best outcomes amongst thrombolytic medications, and is the first choice in many centers, despite its high costs. Alternatives include streptokinase, tenecteplase, reteplase, lanoteplase, and urokinase. • In ED setting, the main reason to initiate therapy with thrombolytics/angioplasty is whether there is ST segment elevation on ECG. Absolute Contraindications to Thrombolytic Therapy • Trauma: Recent head trauma or traumatic CPR • Previous stroke • Recent invasive procedure or surgery • Dissecting aortic aneurysm • Active bleeding or bleeding diathesis Coronary Artery Bypass Grafting • Less often used than the other two in the acute setting. • Benefits of CABG include low rates of event free survival and reintervention free survival • It remains the procedure of choice in patients with severe multivessel disease and complex coronary anatomy.

Dx: 1. ECG a. Markers for ischemia/infarction include: • Peaked T waves: Occur very early and may be missed • ST segment elevation indicates transmural injury and can be diagnostic of an acute infarct. • Q waves: Evidence for necrosis (specific)—Q waves are usually seen late; typically not seen acutely. • T wave inversion is sensitive but not specific. • ST segment depression: Subendocardial injury b. Categories of infarcts • ST segment elevation infarct: Transmural (involves entire thickness of wall); tends to be larger • Non-ST segment elevation infarct: Subendocardial (involves inner one-third to one-half of the wall); tends to be smaller, and presentation is similar to unstable angina—cardiac enzymes differentiate the two 2. Cardiac enzymes—currently the diagnostic gold standard for myocardial injury a. Troponins (Troponin I and T)—most important enzyme test to order • Increases within 3 to 5 hours and returns to normal in 5 to 14 days; reaches a peak in 24 to 48 hours • Greater sensitivity and specificity than creatine kinase-MB (CK-MB) for myocardial injury • Obtain serum levels of either troponin T or troponin I on admission, and again every 8 hours for 24 hours. • Troponin I can be falsely elevated in patients with renal failure. b. CK-MB—less commonly used • Increases within 4 to 8 hours and returns to normal in 48 to 72 hours; reaches a peak in 24 hours • When measured within 24 to 36 hours of onset of chest pain, has greater than 95% sensitivity and specificity • Levels of total CK and CK-MB should be measured on admission and every 8 hours thereafter for 24 hours. Tx: 1. Admit patient to a cardiac monitored floor (CCU) and establish IV access. Give supplemental oxygen and analgesics (nitrates, morphine—see below). 2. Medical therapy a. Aspirin • Antiplatelet agent reduces coronary reocclusion by inhibiting platelet aggregation on top of the thrombus. • Has been shown to reduce mortality and should be part of long-term maintenance therapy b. β-blockers • Block stimulation of HR and contractility to reduce oxygen demand and decrease the incidence of arrhythmias • Reduce remodeling of the myocardium post-MI • Have been shown to reduce mortality and should be part of maintenance therapy c. ACE inhibitors • Initiate within hours of hospitalization if there are no contraindications • Have been shown to reduce mortality and should be part of long-term maintenance therapy d. Statins • Reduce risk of further coronary events • Stabilize plaques and lower cholesterol • The PROVE IT-TIMI 22 trial showed the superiority of starting atorvastatin 80 mg over other statins before discharging a STEMI patient. • Should be part of maintenance therapy e. Oxygen • May limit ischemic myocardial injury f. Nitrates • Dilate coronary arteries (increase supply) • Venodilation (decrease preload and thus demand) • Reduce chest pain, although not as effective as narcotics g. Morphine sulfate • Analgesia • Causes venodilation, which decreases preload and thus oxygen requirements h. Heparin • Initiate in all patients with MI; prevents progression of thrombus; however, has not been shown to decrease mortality • LMWH, specifically enoxaparin, is preferred over unfractionated heparin, as shown in the ExTRACT TIMI 25 trial as well as a recent meta-analysis. Enoxaparin was shown to decrease the risk of another MI versus unfractionated heparin. 3. Revascularization a. Benefit highest when performed early b. Should be considered in all patients c. Revascularization options include thrombolysis, PCI, or CABG—see Clinical Pearl 1-7. • Several studies have shown enhanced survival and lower rates of recurrent MI and intracranial bleeding when PCI performed by skilled personnel is chosen over thrombolysis. The benefit is especially pronounced if the PCI occurs within 90 minutes of arrival at the hospital. For patients with a delayed presentation, fibrinolysis alone may be a better option. • Urgent/emergent CABG is typically performed only in the setting of mechanical complications of an acute MI, cardiogenic shock, life-threatening ventricular arrhythmias, or after failure of PCI. It is almost never performed in the acute setting on a stable patient. d. Clopidogrel—evidence suggests that benefits of clopidogrel is additive to the effects of aspirin. Clopidogrel therapy should be initiated in all patients who undergo PCI and receive a stent. Dual antiplatelet treatment with aspirin and clopidogrel should continue for at least 30 days in patients who receive a bare metal stent, and at least 12 months in patients who receive a drug-eluting stent. 4. Rehabilitation a. Cardiac rehabilitation is a physician-supervised regimen of exercise and risk factor reduction after MI. b. Shown to reduce symptoms and prolong survival

Constrictive pericarditis Causes a. In most patients, the cause is never identified and is idiopathic or related to a previous viral infection. b. Other causes include uremia, radiation therapy, tuberculosis, chronic pericardial effusion, tumor invasion, connective tissue disorders, and prior surgery involving the pericardium.

Dx: 1. ECG a. Nonspecific changes such as low QRS voltages, generalized T wave flattening or inversion, left atrial abnormalities b. Atrial fibrillation is more often seen in advanced disease but overall occurs in fewer than half of all patients. 2. Echocardiogram a. Increased pericardial thickness is seen in about half of all patients. b. A sharp halt in ventricular diastolic filling and atrial enlargement can also be seen. 3. CT scan and MRI may also show pericardial thickening and calcifications, and can aid greatly in the diagnosis. 4. Cardiac catheterization a. Elevated and equal diastolic pressures in all chambers b. Ventricular pressure tracing shows a rapid y descent, which has been described as a dip and plateau or a "square root sign." Tx: 1. Treat the underlying condition. 2. Diuretics may be extremely helpful in treating fluid overload symptoms. 3. Monitor and treat for any coagulopathy.

Atrial fibrillation The treatment of AFib and atrial flutter are similar. There are three main goals: • Control ventricular rate (metoprolol) • Restore normal sinus rhythm (cardioversion) • Assess need for anticoagulation (heparin-warfarin) Cardioversion: • Delivery of a shock that is in synchrony with the QRS complex: Purpose is to terminate certain dysrhythmias such as PSVT or VT; an electric shock during T wave can cause Vfib, so the shock is timed not to hit the T wave. • Indications: AFib, atrial flutter, VT with a pulse, SVT Defibrillation: • Delivery of a shock that is not in synchrony with the QRS complex: Purpose is to convert a dysrhythmia to normal sinus rhythm • Indications: VFib, VT without a pulse Automatic Implantable Defibrillator: • Device that is surgically placed: When it detects a lethal dysrhythmia, it delivers an electric shock to defibrillate. It delivers a set number of shocks until the dysrhythmia is terminated. • Indications: VFib and/or VT that is not controlled by medical therapy

Dx: 1. ECG findings: Irregularly irregular rhythm (irregular RR intervals and excessively rapid series of tiny, erratic spikes on ECG with a wavy baseline and no identifiable P waves) Tx: 1. Acute AFib in a hemodynamically unstable patient: Immediate electrical cardioversion to sinus rhythm 2. Acute AFib in a hemodynamically stable patient: a. Rate control • Determine the pulse in a patient with AFib. If it is too rapid, it must be treated. Target rate is 60 to 100 bpm. • β-blockers are preferred. Calcium channel blockers are an alternative. • If left ventricular systolic dysfunction is present, consider digoxin or amiodarone (useful for rhythm control). b. Cardioversion to sinus rhythm (after rate control is achieved) • Candidates for cardioversion include those who are hemodynamically unstable, those with worsening symptoms, and those who are having their first ever case of AFib. • Electrical cardioversion is preferred over pharmacologic cardioversion. Attempts should be made to control ventricular rate before attempting DC cardioversion. • Use pharmacologic cardioversion only if electrical cardioversion fails or is not feasible: Parenteral ibutilide, procainamide, flecainide, sotalol, or amiodarone are choices. c. Anticoagulation to prevent embolic cerebrovascular accident (CVA) • If AFib present >48 hours (or unknown period of time), risk of embolization during cardioversion is significant (2% to 5%). Anticoagulate patients for 3 weeks before and 4 weeks after cardioversion. • An INR of 2 to 3 is the anticoagulation goal range. • To avoid waiting 3 weeks for anticoagulation, obtain a transesophageal echocardiogram (TEE) to image the left atrium. If no thrombus is present, start IV heparin and perform cardioversion within 24 hours. Patients still require 4 weeks of anticoagulation after cardioversion. 3. Chronic AFib a. Rate control with a β-blocker or calcium channel blocker b. Anticoagulation • Patients with "lone" AFib (i.e., AFib in the absence of underlying heart disease or other cardiovascular risk factors) under age 60 do not require anticoagulation because they are at low risk for embolization (aspirin may be appropriate). • Treat all other patients with chronic anticoagulation (warfarin).

Multifocal atrial tachycardia

Dx: 1. ECG findings: Variable P wave morphology and variable PR and RR intervals. At least three different P wave morphologies are required to make an accurate diagnosis. 2. The diagnosis of wandering atrial pacemaker is identical except that the heart rate is between 60 and 100 bpm (i.e., not tachycardic). 3. Can also be diagnosed by use of vagal maneuvers or adenosine to show AV block without disrupting the atrial tachycardia Tx: 1. Improving oxygenation and ventilation (strong association between MAT and COPD). 2. If left ventricular function is preserved, acceptable treatments include calcium channel blockers, β-blockers, digoxin, amiodarone, IV flecainide, and IV propafenone. 3. If LV function is not preserved, use digoxin, diltiazem, or amiodarone. Electrical cardioversion is ineffective and should not be used.

Atrial flutter

Dx: 1. ECG provides a saw-tooth baseline, with a QRS complex appearing after every second or third "tooth" (P wave). Saw-tooth flutter waves are best seen in the inferior leads (II, III, aVF). Tx: 1. Similar to treatment for AFib • Control ventricular rate (metoprolol) • Restore normal sinus rhythm (cardioversion); electrical or pharmacological with amiodarone • Assess need for anticoagulation (heparin-warfarin)

Coarctation of the aorta Signs Midsystolic murmur heard best over the back

Dx: 1. ECG shows LVH. 2. CXR a. Notching of the ribs b. "Figure 3" appearance due to indentation of the aorta at site of coarctation, dilation before and after stenosis Tx: 1. Standard treatment involves surgical decompression. 2. Percutaneous balloon aortoplasty is also an option in selected cases.

Acute pericarditis Causes a. Idiopathic (probably postviral): Most cases of idiopathic pericarditis are presumed to be postviral, usually preceded by a recent flulike illness or by upper respiratory or GI symptoms. b. Infectious: Viral (e.g., Coxsackievirus echovirus, adenovirus, EBV, influenza, HIV, hepatitis A or B), bacterial (tuberculosis), fungal, toxoplasmosis c. Acute MI (first 24 hours after MI) d. Uremia e. Collagen vascular diseases (e.g., SLE, scleroderma, rheumatoid arthritis, sarcoidosis) f. Neoplasm—especially Hodgkin lymphoma, breast, and lung cancers g. Drug-induced lupus syndrome (procainamide, hydralazine) h. After MI: (Dressler's syndrome)—usually weeks to months after MI i. After surgery—postpericardiotomy syndrome j. Amyloidosis k. Radiation l. Trauma

Dx: 1. ECG shows four changes in sequence a. Diffuse S-T elevation and PR depression (PR depression is specific ECG finding in acute pericarditis) b. ST segment returns to normal—typically 1 week c. T wave inverts—does not occur in all patients d. T wave returns to normal 2. Echocardiogram if pericarditis with effusion is suspected, but echocardiogram is often normal Tx: 1. Most cases are self-limited, resolve in 2 to 6 weeks. 2. Treat the underlying cause if known. 3. NSAIDs are the mainstay of therapy (for pain and other systemic symptoms). Colchicine is often used. 4. Glucocorticoids may be tried if pain does not respond to NSAIDs, but should be avoided if at all possible. 5. Relatively uncomplicated cases can be treated as an outpatient. However, patients with more worrisome symptoms such as fever and leukocytosis and patients with worrisome features such as pericardial effusion or tamponade should be hospitalized.

Dilated cardiomyopathy 1. Up to 50% of cases are idiopathic. 2. Other causes include: a. CAD (with prior MI) is a common cause. b. Toxic: Alcohol, doxorubicin, Adriamycin c. Metabolic: Thiamine or selenium deficiency, hypophosphatemia, uremia d. Infectious: Viral, Chagas' disease, Lyme disease, HIV e. Thyroid disease: Hyperthyroidism or hypothyroidism f. Peripartum cardiomyopathy g. Collagen vascular disease: SLE, scleroderma h. Prolonged, uncontrolled tachycardia i. Catecholamine induced: Pheochromocytoma, cocaine j. Familial/genetic

Dx: 1. ECG, CXR, and echocardiogram results consistent with CHF 2. Genetic testing may be warranted if there is a family history of DCM and no other cause can be identified. Tx: 1. Similar to treatment of CHF: Digoxin, diuretics, vasodilators, and cardiac transplantation 2. Remove the offending agent if possible. 3. Anticoagulation should be considered because patients are at increased risk of embolization Standard treatment of CHF includes a loop diuretic, ACE inhibitor, and beta-blocker. Depending on severity and patient factors, other medications such as digoxin, hydralazine/nitrate, spironolactone may be added.

Premature atrial complexes

Dx: 1. ECG, look for early P waves that differ in morphology from the normal sinus P wave (because these P waves originate within the atria and not the sinus node). 2. QRS complex is normal because conduction below the atria is normal. There is usually a pause before the next sinus P wave. Tx: 1. Usually asymptomatic and do not require treatment. Monitor for increased frequency. 2. If symptomatic (e.g., palpitations), β-blockers may be helpful.

Ventricular septal defect Signs a. Harsh, blowing holosystolic murmur with thrill • At fourth left intercostal space • Murmur decreases with Valsalva and handgrip • The smaller the defect, the louder the holosystolic murmur b. Sternal lift (RV enlargement) c. As PVR increases, the pulmonary component of S2 increases in intensity. d. Aortic regurgitation may be seen in some patients.

Dx: 1. ECG: Biventricular hypertrophy predominates when PVR is high. 2. CXR a. Enlargement of the pulmonary artery b. Enlargement of cardiac silhouette: As PVR increases (and left-to-right shunt decreases), heart size decreases, but pulmonary artery size increases. 3. Echocardiogram shows the septal defect. Tx: 1. Endocarditis prophylaxis is important but is NOT currently recommended for patients with uncomplicated VSD and no history of endocarditis. 2. Surgical repair is indicated if the pulmonary flow to systemic flow ratio is greater than 1.5:1 or 2:1, as well as for patients with infective endocarditis. 3. For the asymptomatic patient with a small defect, surgery is not indicated.

Wolff-Parkinson-White syndrome

Dx: 1. ECG: Narrow complex tachycardia, a short P-R interval, and a delta wave (upward deflection seen before the QRS complex) Tx: 1. Radiofrequency catheter ablation of one arm of the reentrant loop (i.e., of the accessory pathway) is an effective treatment. Medical options include procainamide or quinidine. 2. Avoid drugs active on the AV node (e.g., digoxin, verapamil) because they may accelerate conduction through the accessory pathway. Type IA or IC antiarrhythmics are better choices.

Ventricular fibrillation 1. Ischemic heart disease is the most common cause. 2. Antiarrhythmic drugs, especially those that cause torsades de pointes (prolonged QT intervals) 3. AFib with a very rapid ventricular rate in patients with Wolff-Parkinson-White syndrome Recurrence a. If VFib is not associated with acute MI, recurrence rate is high (up to 30% within the first year). These patients require chronic therapy: Either prophylactic antiarrhythmic therapy (amiodarone) or implantation of an automatic defibrillator. b. If VFib develops within 48 hours of an acute MI, long-term prognosis is favorable and the recurrence rate is low (2% at 1 year). Chronic therapy is not required in these patients. c. Fatal if untreated.

Dx: 1. ECG: No atrial P waves can be identified. 2. No QRS complexes can be identified. 3. In sum, no waves can be identified; there is a very irregular rhythm. Tx: 1. This is a medical emergency! Immediate defibrillation and CPR are indicated. a. Initiate unsynchronized DC cardioversion immediately. If the equipment is not ready, start CPR until it is. b. Give up to three sequential shocks to establish another rhythm; assess the rhythm between each. 2. If VF persists: a. Continue CPR. b. Intubation may be indicated. c. Epinephrine (1 mg IV bolus initially, and then every 3 to 5 minutes)—this increases myocardial and cerebral blood flow and decreases the defibrillation threshold. d. Attempt to defibrillate again 30 to 60 seconds after first epinephrine dose. 3. Other options if above procedures fail (refractory VFib): a. IV amiodarone followed by shock—new ACLS guidelines recommend the use of amiodarone over other antiarrhythmic agents in refractory VFib. b. Lidocaine, magnesium, and procainamide are alternative second-line treatments. c. Sodium bicarbonate is no longer recommended. 4. If cardioversion is successful: a. Maintain continuous IV infusion of the effective antiarrhythmic agent. IV amiodarone has been shown to be the most effective. b. Implantable defibrillators have become the mainstay of chronic therapy in patients at continued risk for VF. Long-term amiodarone therapy is an alternative. 5. Drugs cannot convert VFib by themselves. Defibrillation is key. Defibrillation generally does not work for asystole. Try transcutaneous pacing instead.

Ventricular tachycardia Causes a. CAD with prior MI is the most common cause b. Active ischemia, hypotension c. Cardiomyopathies d. Congenital defects e. Prolonged QT syndrome f. Drug toxicity

Dx: 1. ECG: Wide and bizarre QRS complexes 2. QRS complexes may be monomorphic or polymorphic. a. In monomorphic VT, all QRS complexes are identical. b. In polymorphic VT, the QRS complexes are different. 3. Unlike PSVT, VT does not respond to vagal maneuvers or adenosine. Tx: 1. Identify and treat reversible causes. 2. Sustained VT a. Hemodynamically stable patients with mild symptoms and systolic BP > 90 —pharmacologic therapy • New advanced cardiac life support (ACLS) guidelines recommend IV amiodarone, IV procainamide, or IV sotalol over IV lidocaine or IV bretylium. b. Hemodynamically unstable patients or patients with severe symptoms • Immediate synchronous DC cardioversion • Follow with IV amiodarone to maintain sinus rhythm c. Ideally, all patients with sustained VT should undergo placement of an ICD, unless EF is normal (then consider amiodarone). 3. Nonsustained VT a. If no underlying heart disease and asymptomatic, do not treat. These patients are not at increased risk of sudden death. b. If the patient has underlying heart disease, a recent MI, evidence of left ventricular dysfunction, or is symptomatic, order an electrophysiologic study: If it shows inducible, sustained VT, ICD placement is appropriate. c. Pharmacologic therapy is second-line treatment. However, amiodarone has the best results of all of the antiarrhythmic agents.

Cardiogenic shock General characteristics 1. Occurs when heart is unable to generate a cardiac output sufficient to maintain tissue perfusion 2. Can be defined as a systolic BP < 90 with urine output <20 mL/hr and adequate left ventricular filling pressure Note that jugular venous pulse/PCWP is only elevated in cardiogenic shock. Intraaortic Balloon Pump (IABP) • A device that gives "mechanical support" to a failing heart—it works opposite to the normal pumping action of the heart, that is, it serves to "pump" during diastole and "relax" during systole. • A balloon catheter is positioned in the descending thoracic aorta just distal to subclavian artery. It facilitates ventricular emptying by deflating just before the onset of systole (reducing afterload) and increases coronary perfusion by inflating at the onset of diastole (increasing diastolic pressure). • The net effect is enhanced myocardial oxygenation and increased cardiac output. • Indications are angina refractory to medical therapy, mechanical complications of MI, cardiogenic shock, low cardiac output states, and as a bridge to surgery in severe aortic stenosis.

Dx: 1. ECG—ST segment elevation suggesting acute MI or arrhythmia are the most common findings. 2. Echocardiogram—can diagnose a variety of mechanical complications of MI, identify valve disease, estimate EF, look for pericardial effusion, etc. 3. Hemodynamic monitoring with a Swan-Ganz catheter may be indicated: PCWP, pulmonary artery pressure, cardiac output, cardiac index, SVR—keep cardiac output >4 L/min, cardiac index >2.2, PCWP < 18 mm Hg Tx: 1. ABCs 2. Identify and treat underlying cause a. Acute MI • Standard treatment aspirin, heparin (see MI section) • Aggressive management, that is, emergent revascularization with PCI (or CABG), has been shown to improve survival. b. If cardiac tamponade, pericardiocentesis/surgery c. Surgical correction of valvular abnormalities d. Treatment of arrhythmias 3. Vasopressors a. Dopamine is often the initial drug used. b. Dobutamine may be used in combination with dopamine to further increase cardiac output. c. Norepinephrine or phenylephrine may be used in severe or resistant cases. 4. Afterload-reducing agents such as nitroglycerin or nitroprusside are typically not used initially because they aggravate hypotension. They may be used later with vasopressors. 5. IV fluids are likely to be harmful if left ventricular pressures are elevated. Patients may in fact need diuretics/lasix 6. While still controversial, IABPs are often used for hemodynamic support; Effects include: a. Decreased afterload b. Increased cardiac output c. Decreased myocardial oxygen demand

Pericardial effusion

Dx: 1. Echocardiogram a. Imaging procedure of choice: Confirms the presence or absence of a significant effusion b. Most sensitive and specific method of determining whether pericardial fluid is present; can show as little as 20 mL of fluid c. Should be performed in all patients with acute pericarditis to rule out an effusion 2. CXR a. CXR shows enlargement of cardiac silhouette when >250 mL of fluid has accumulated. b. Cardiac silhouette may have prototypical "water bottle" appearance. c. An enlarged heart without pulmonary vascular congestion suggests pericardial effusion. 3. ECG a. Shows low QRS voltages and T wave flattening but should not be used to diagnose pericardial effusion b. Electrical alternans suggests a massive pericardial effusion and tamponade. 4. CT scan or MRI—very accurate, but often unnecessary given the accuracy of an echocardiogram 5. Pericardial fluid analysis—may clarify the cause of the effusion a. Order protein and glucose content, cell count and differential, cytology, specific gravity, hematocrit, Gram stain, acid-fast stains, fungal smear, cultures, LDH content Tx: 1. Depends on patient's hemodynamic stability 2. Pericardiocentesis is not indicated unless there is evidence of cardiac tamponade. Analysis of pericardial fluid can be useful if the cause of the effusion is unknown. 3. If the effusion is small and clinically insignificant, a repeat echocardiogram in 1 to 2 weeks is appropriate.

Restrictive cardiomyopathy

Dx: 1. Echocardiogram a. Thickened myocardium and possible systolic ventricular dysfunction b. Increased right atrium (RA) and left atrium (LA) size with normal LV and RV size c. In amyloidosis, myocardium appears brighter or may have a sparkled appearance. 2. ECG: Low voltages or conduction abnormalities, arrhythmias, AFib 3. Endomyocardial biopsy may be diagnostic. Tx: 1. Treat underlying disorder a. Hemochromatosis: Phlebotomy or deferoxamine b. Sarcoidosis: Glucocorticoids c. Amyloidosis: No treatment available d. Give digoxin if systolic dysfunction is present (except in patients with cardiac amyloidosis, who have increased incidence of digoxin toxicity). 2. Use diuretics and vasodilators (for pulmonary and peripheral edema) cautiously, because a decrease in preload may compromise cardiac output.

Hypertrophic cardiomyopathy

Dx: 1. Echocardiogram establishes the diagnosis 2. Clinical diagnosis and family history Tx: 1. Asymptomatic patients generally do not need treatment, but this is controversial. No studies have shown any alteration in the prognosis with therapy, so treatment generally focuses on reducing symptoms. 2. All patients should avoid strenuous exercise. 3. Symptomatic patients a. β-Blockers should be the initial drug used in symptomatic patients; they reduce symptoms by improving diastolic filling (as HR decreases, duration in diastole increases), and also reduce myocardial contractility and thus oxygen consumption. b. Calcium channel blockers (verapamil) • Can be used if patient is not responding to β-blocker • Reduce symptoms by similar mechanism as β-blockers c. Diuretics can be used if fluid retention occurs. d. If AFib is present, treat accordingly (see Atrial Fibrillation). e. Surgery • Myomectomy has a high success rate for relieving symptoms. It involves the excision of part of the myocardial septum. It is reserved for patients with severe disease. • Mitral valve replacement is now rarely performed. f. Pacemaker implantation has had variable results.

Mitral valve prolapse Signs a. Midsystolic or late systolic click(s) b. Mid-to-late systolic murmur c. Some patients have midsystolic click without murmur; others may have the murmur without click. d. Standing and the Valsalva maneuver increase murmur and click because these maneuvers reduce LV chamber size, allowing the click and murmur to occur earlier in systole. e. Squatting decreases murmur and click because it increases LV chamber size, thus delaying the onset of the click and murmur.

Dx: 1. Echocardiogram is the most useful 2. Most patients are asymptomatic, so diagnosis is typically made on the basis of the murmur and echocardiogram alone. Tx: 1. If patient is asymptomatic, reassurance. There is some association between MVP and anxiety, so all patients should be reassured about the benign nature of this condition. 2. For chest pain, β-blockers have been useful, but they are unlikely to be required. 3. Surgery is rarely required. The condition is generally benign.

Tetralogy of Fallot (Ventricular septal defect, right ventricular hypertrophy, pulmonary artery stenosis, and overriding aorta) Signs Murmur is typically crescendo-decrescendo in nature and heard best at the left upper sternal border.

Dx: 1. Echocardiography is the diagnostic modality of choice. This test can clearly define the four abnormalities as well as provide important information about aortic arch anatomy. 2. EKG may show enlarged RA and RV. 3. Chest x-ray classically shows boot-shaped heart. 4. Cardiac catheterization may be required in some patients to fully define the anatomy. Tx: 1. Treatment is surgical. Most patients have surgery within the first year of life. Twenty year survival rates after surgery are above 80%. The most common causes of death are sudden cardiac death and heart failure. 2. Complications after surgery include arrhythmias, pulmonary regurgitation, residual outflow obstruction, and heart failure.

Variant Prinzmetal's angina

Dx: 1. Episodes of angina occur at rest and are associated with ventricular dysrhythmias, some of which may be life threatening. The angina classically occurs at night. 2. Hallmark is transient ST segment elevation (not depression) on ECG during chest pain, which represents transmural ischemia 3. Coronary angiography is a definitive test—displays coronary vasospasm when the patient is given IV ergonovine (to provoke chest pain). Tx: 1. Vasodilators—calcium channel blockers and nitrates have been proven to be helpful. 2. Risk factor modification including smoking cessation and lipid lowering is also indicated where appropriate.

Hypertensive emergency

Dx: 1. Hypertensive emergency: Systolic BP > 220 and/or diastolic BP > 120 in addition to end-organ damage—immediate treatment is indicated. 2. Elevated BP levels alone without end-organ damage—referred to as hypertensive urgency. Hypertensive urgencies rarely require emergency therapy and can be managed with attempts to lower BP over a period of 24 hours. 3. Assess for papilledema, altered mental status or intracranial bleed, renal failure/hematuria, pulmonary edema, and USA/MI/CHF/aortic dissection Tx: 1. Hypertensive emergencies a. Reduce mean arterial pressure by 25% in 1 to 2 hours. The goal is not to immediately achieve normal BP, but to get the patient out of danger, then reduce BP gradually. b. If severe (diastolic pressure >130) or if hypertensive encephalopathy is present, IV agents such as hydralazine, esmolol, nitroprusside, labetalol, or nitroglycerin are appropriate. c. In patients who are in less immediate danger, oral agents are appropriate. Options include captopril, clonidine, labetalol, nifedipine, and diazoxide. 2. Hypertensive urgencies: BP should be lowered within 24 hours using oral agents. 3. If a Patient Presents With Severe Headache and Markedly Elevated BP • The first step is to lower the BP with an antihypertensive agent. • The second step is to order a CT scan of the head to rule out intracranial bleeding (subarachnoid hemorrhage is in the differential diagnosis for severe headache). • If the CT scan is negative, one may proceed to a lumbar puncture.

Hypovolemic shock Causes a. Hemorrhage • Trauma • GI bleeding • Retroperitoneal b. Nonhemorrhagic • Voluminous vomiting • Severe diarrhea • Severe dehydration for any reason • Burns • Third-space losses in bowel obstruction • Compensatory mechanisms begin to fail when more than 20% to 25% of blood volume is lost. • If CVP is low, hypovolemic shock is most likely present. Class; Loss; HR(↑); SBP(↓); PP(↓); CR(↓); RR(↑); CNS; UO(↓) I 10%-15%, Normal for all II 20%-30%, >100, Nl, Dec, Delayed, Mild tachypnea, Anxious, 20-30 mL/hr III 30%-40%, >120 weak, Dec, Dec, Delayed, Marked tachypnea, Confused, 20 mL/hr IV >40%, >140 non-palpable, Marked dec, Marked dec, Absent, Marked tachypnea, Lethargic/coma, none

Dx: 1. If the diagnosis is unclear from the patient's vital signs and clinical picture, a central venous line or a pulmonary artery catheter can give invaluable information for hemodynamic monitoring: decreased CVP, PCWP, cardiac output, increased SVR 2. Monitoring urine output is most important for treatment efficacy Tx: 1. Airway and breathing—patients in severe shock and circulatory collapse generally require intubation and mechanical ventilation. 2. Circulation a. If hemorrhage is the cause, apply direct pressure. b. IV hydration • Patients with class I shock usually do not require fluid resuscitation. Patients with class II shock benefit from fluids, and patients with classes III and IV require fluid resuscitation. • Give fluid bolus followed by continuous infusion and reassess. • The hemodynamic response to this treatment guides further resuscitative effort. c. For nonhemorrhagic shock, blood is not necessary. Crystalloid solution with appropriate electrolyte replacement is adequate.

Myocarditis Many possible causes, including viruses (e.g., Coxsackie, parvovirus B19, human herpes virus-6), bacteria (e.g., group A streptococcus in rheumatic fever, Lyme disease, mycoplasma, etc.), SLE, medications (e.g., sulfonamides); can also be idiopathic

Dx: 1. Look for elevations in cardiac enzyme levels 2. Erythrocyte sedimentation rate Tx: 1. Treatment is supportive. 2. Treat underlying causes if possible, and treat any complications.

Rheumatic heart disease

Dx: 1. Major criteria (need 2 JONES) a. Migratory polyarthritis b. Erythema marginatum c. Cardiac involvement (e.g., pericarditis, CHF, valve dz) d. Sydenham chorea e. Subcutaneous nodules 2. Minor criteria (need 2 also) a. Fever b. Elevated erythrocyte sedimentation rate c. Polyarthralgias d. Prior history of rheumatic fever e. Prolonged PR interval f. Evidence of preceding streptococcal infection Tx: 1. Treat streptococcal pharyngitis with penicillin or erythromycin to prevent rheumatic fever. 2. Acute rheumatic fever is treated with NSAIDs. C-reactive protein is used to monitor treatment. 3. Patients with a history of rheumatic fever should receive antibiotic prophylaxis with erythromycin or amoxicillin for dental/GI/GU procedures. 4. Treat the valvular pathology of rheumatic heart disease (MS, aortic or tricuspid as well).

2nd degree type 1 AV block

Dx: 1. Mobitz type I (Wenckebach) a. Characterized by progressive prolongation of PR interval until a P wave fails to conduct b. Site of block is usually within the AV node Tx: 1. Benign condition that does not require treatment

2nd degree type 2 AV block

Dx: 1. Mobitz type II a. P wave fails to conduct suddenly, without a preceding PR prolongation; therefore, QRS drops suddenly. b. Often progresses to complete heart block c. Site of block is within the His-Purkinje system Tx: 1. Pacemaker implantation is necessary.

Stable angina pectoris Standard of care for stable angina is aspirin and a beta blocker (only ones that lower mortality), and nitrates for chest pain. Types of Stress Tests: detecting ischemia -Exercise tolerance test = ST segment depression -Exercise or dobutamine echocardiogram = Wall motion abnormalities -Exercise or dipyridamole thallium = Decreased uptake of the nuclear isotope during exercise

Dx: 1. Note that physical examination in most patients with CAD is normal. 2. Resting ECG a. Usually normal in patients with stable angina b. Q waves are consistent with a prior myocardial infarction. c. If ST segment or T wave abnormalities are present during an episode of chest pain, then treat as for unstable angina. 3. Stress test—useful for patients with an intermediate pretest probability of CAD based upon age, gender, symptoms. For patients with normal resting ECG, determine whether the patient is capable of performing treadmill exercise. If so, proceed to an exercise stress test. a. Stress ECG • Test involves recording ECG before, during, and after exercise on a treadmill. • 75% sensitive if patients are able to exercise sufficiently to increase heart rate to 85% of maximum predicted value for age. A person's maximum heart rate is calculated by subtracting age from 220 (220 − age). • Exercise-induced ischemia results in subendocardial ischemia, producing ST segment depression. So the detection of ischemia on a stress test is based on presence of ST segment depression. • Other positive findings include onset of heart failure or ventricular arrhythmia during exercise or hypotension. • Patients with a positive stress test result should undergo cardiac catheterization (A stress test is generally considered positive if the patient develops any of the following during exercise: ST segment depression, chest pain, hypotension, or significant arrhythmias.) b. Stress echocardiography • Performed before and immediately after exercise. Exercise-induced ischemia is evidenced by wall motion abnormalities (e.g., akinesis or dyskinesis) not present at rest. • Favored by many cardiologists over stress ECG. It is more sensitive in detecting ischemia, can assess LV size and function, can diagnose valvular disease, and can be used to identify CAD in the presence of preexisting ECG abnormalities (see Clinical Pearl 1-2). • Again, patients with a positive test result should undergo cardiac catheterization. c. Information gained from a stress test can be enhanced by stress myocardial perfusion imaging after IV administration of a radioisotope such as thallium 201 during exercise. • Viable myocardial cells extract the radioisotope from the blood. No radioisotope uptake means no blood flow to an area of the myocardium. • It is important to determine whether the ischemia is reversible, that is, whether areas of hypoperfusion are perfused over time as blood flow eventually equalizes. Areas of reversible ischemia may be rescued with percutaneous coronary intervention (PCI) or coronary artery bypass graft (CABG). Irreversible ischemia, however, indicates infarcted tissue that cannot be salvaged. • Perfusion imaging increases the sensitivity and specificity of exercise stress tests, but is also more expensive, subjects the patient to radiation, and is often not helpful in the presence of a left bundle branch block. 4. If the patient cannot exercise, perform a pharmacologic stress test. a. IV adenosine, dipyridamole, or dobutamine can be used. The cardiac stress induced by these agents takes the place of exercise. This can be combined with an ECG, an echocardiogram, or nuclear perfusion imaging. b. IV adenosine and dipyridamole cause generalized coronary vasodilation. Since diseased coronary arteries are already maximally dilated at rest to increase blood flow, they receive relatively less blood flow when the entire coronary system is pharmacologically vasodilated. c. Dobutamine increases myocardial oxygen demand by increasing heart rate, blood pressure, and cardiac contractility. 5. Holter monitoring (ambulatory ECG) can be useful in detecting silent ischemia (i.e., ECG changes not accompanied by symptoms). The Holter monitor is also used for evaluating arrhythmias, heart rate variability, and to assess pacemaker and implantable cardioverter-defibrillator (ICD) function. a. Continuously examines patient's cardiac rhythm over 24 to 72 hours during normal activity b. Useful for evaluating unexplained syncope and dizziness as well 6. Cardiac catheterization with coronary angiography (There are many indications for cardiac catheterization (generally performed when revascularization or other surgical intervention are being considered): • After a positive stress test • In a patient with angina in any of the following situations: When noninvasive tests are nondiagnostic, angina that occurs despite medical therapy, angina that occurs soon after MI, and any angina that is a diagnostic dilemma. • If patient is severely symptomatic and urgent diagnosis and management are necessary • For evaluation of valvular disease, and to determine the need for surgical intervention) a. Coronary angiography—definitive test for CAD. Often performed with concurrent PCI or for patients being considered for revascularization with CABG. b. Contrast is injected into coronary vessels to visualize any stenotic lesions. This defines the location and extent of coronary disease. c. Angiography is the most accurate test for detecting CAD. d. If coronary artery disease is severe (e.g., left main or three-vessel disease), refer patient for surgical revascularization (CABG). Tx: Risk factor modification a. Smoking cessation cuts coronary heart disease (CHD) risk in half by 1 year after quitting. b. HTN—vigorous BP control reduces the risk of CHD, especially in diabetic patients. c. Hyperlipidemia—reduction in serum cholesterol with lifestyle modifications and HMG-CoA reductase inhibitors reduce CHD risk. d. DM—type II diabetes is considered to be a cardiovascular heart disease equivalent, and strict glycemic control should be strongly emphasized. e. Obesity—weight loss modifies other risk factors (diabetes, HTN, and hyperlipidemia) and provides other health benefits. f. Exercise is critical; it minimizes emotional stress, promotes weight loss, and helps reduce other risk factors. g. Diet: Reduce intake of saturated fat (<7% total calories) and cholesterol (<200 mg/day). 2. Medical therapy a. Aspirin • Indicated in all patients with CAD • Decreases morbidity—reduces risk of MI b. β-blockers—block sympathetic stimulation of heart. First-line choices include atenolol and metoprolol. • Reduce HR, BP, and contractility, thereby decreasing cardiac work (i.e., β-blockers lower myocardial oxygen consumption) • Have been shown to reduce the frequency of coronary events c. Nitrates—cause generalized vasodilation • Relieve angina; reduce preload myocardial oxygen demand • May prevent angina when taken before exertion • Effect on prognosis is unknown; main benefit is symptomatic relief • Can be administered orally, sublingually, transdermally, intravenously, or in paste form. For chronic angina, oral or transdermal patches are used. For acute coronary syndromes (see below), either sublingual, paste, or IV forms are used. d. Calcium channel blockers • Cause coronary vasodilation and afterload reduction, in addition to reducing contractility • Now considered a secondary treatment when β-blockers and/or nitrates are not fully effective. None of the calcium channel blockers have been shown to lower mortality in CAD. In fact, they may increase mortality because they raise heart rates. Do not routinely use these drugs in CAD. e. If congestive heart failure (CHF) is also present, treatment with ACE inhibitors and/or diuretics may be indicated as well. 3. Revascularization a. May be preferred for high-risk patients, although there is some controversy whether revascularization is superior to medical management for a patient with stable angina and stenosis >70% b. Two methods—PCI and CABG c. Revascularization does not reduce incidence of MI, but does result in significant improvement symptoms. 4. Management decisions (general guidelines)—risk factor modification and aspirin are indicated in all patients. Manage patients according to overall risk: a. Mild disease (normal EF, mild angina, single-vessel disease) • Nitrates (for symptoms and as prophylaxis) and a β-blocker are appropriate. • Consider calcium channel blockers if symptoms continue despite nitrates and β-blockers. b. Moderate disease (normal EF, moderate angina, two-vessel disease) • If the above regimen does not control symptoms, consider coronary angiography to assess suitability for revascularization (either PCI/angioplasty or CABG). c. Severe disease (decreased EF, severe angina, and three-vessel/left main or left anterior descending disease) • Coronary angiography and consider for CABG

Pulseless electrical activity PEA

Dx: 1. Occurs when electrical activity is on the monitor but there are no pulses (even with Doppler), and carries a grim prognosis Tx: 1. Treat possible causes (hypoxia, hypovolemia, hypotension, hyperkalemia, tamponade, tension pneumothorax, massive PE, and so on) 2. Medicate according to ACLS guidelines.

1st degree AV block

Dx: 1. PR interval is prolonged (>0.20 sec). 2. A QRS complex follows each P wave. 3. Delay is usually in the AV node. Tx: 1. Benign condition that does not require treatment

Unstable angina pectoris Thrombolysis in Myocardial Infarction (TIMI) Risk Score • A prognostication scheme that categorizes risk of death and ischemic events in patients with unstable angina/non-ST segment elevation MI. • One point given for each of the following: • Age >65 years • More than three risk factors for CAD • Known CAD (stenosis >50%) • At least two episodes of severe angina in past 24 hours • Aspirin use in past 7 days • Elevated serum cardiac enzymes • ST changes >0.5 mm • Risk of major adverse cardiac events (all-cause mortality, new or recurrent MI or severe recurrent ischemia requiring urgent revascularization) at 14 days is based on number of points above: • 0-1 point: 5% • 2 points: 8% • 3 points: 13% • 4 points: 20% • 5 points: 26% • 6-7 points: 41%

Dx: 1. Perform a diagnostic workup to exclude MI. 2. Patients with USA have a higher risk of adverse events during stress testing. These patients should be stabilized with medical management before stress testing or should undergo cardiac catheterization initially. Tx: 1. Hospital admission on a floor with continuous cardiac monitoring. Establish IV access and give supplemental oxygen. Provide pain control with nitrates (below) and morphine. 2. Aggressive medical management is indicated—treat as in MI except for fibrinolysis. a. Aspirin b. Clopidogrel—shown to reduce the incidence of MI in patients with USA compared with aspirin alone in the CURE trial. This benefit persists whether the patient undergoes revascularization with PCI or not. Patients presenting with USA should generally be treated with aspirin and clopidogrel for 9 to 12 months, in accordance with the CURE trial. This may be altered however, according to the bleeding risk of each patient. c. Beta-blockers—first-line therapy if there are no contraindications d. Low-molecular-weight heparin (LMWH) is superior to unfractionated heparin. Goal is to prevent progression or development of a clot. • Should be continued for at least 2 days • Keep PTT at 2 to 2.5 times normal if using unfractionated heparin; PTT not followed with LMWH • Enoxaparin is the drug of choice based on clinical trials (see Quick Hit on ESSENCE trial). e. Nitrates are first-line therapy. f. Oxygen if patient is hypoxic g. Glycoprotein IIb/IIIa inhibitors (abciximab, tirofiban) can be helpful adjuncts in USA, especially if patient is undergoing PTCA or stenting. h. Morphine is controversial—provides good pain relief but may mask worsening symptoms i. Replacement of deficient electrolytes, especially K+ and Mg2+ 3. Cardiac catheterization/revascularization a. More than 90% of patients improve with the above medical regimen within 1 to 2 days. b. The choice of invasive management (early catheterization/revascularization within 48 hours) versus conservative management (catheterization/revascularization only if medical therapy fails) is controversial. • No study has shown a significant difference in outcomes between these two approaches. • If patient responds to medical therapy, perform a stress ECG to assess need for catheterization/revascularization. Many patients with USA that is controlled with medical therapy eventually require revascularization. • If medical therapy fails to improve symptoms and/or ECG changes indicative of ischemia persist after 48 hours, then proceed directly to catheterization/revascularization. Additional indications for PCI include hemodynamic instability, ventricular arrhythmias, and new mitral regurgitation or new septal defect. • The TIMI risk score can be used to guide the decision on conservative versus more aggressive treatment—see Clinical Pearl 1-5. 4. After the acute treatment a. Continue aspirin (or other antiplatelet therapy), β-blockers (atenolol or metoprolol), and nitrates b. Reduce risk factors • Smoking cessation, weight loss • Treat diabetes, HTN • Treat hyperlipidemia—patients with USA (or non-ST segment elevation MI) with elevated LDL cholesterol should be started on an HMG-CoA reductase inhibitor. Clinical trials of statins have shown the efficacy of such therapy for secondary prevention in CAD (The CARE trial: Patients with prior history of MI were randomized to treatment with statins or placebo. The statin group had a reduced risk of death (by 24%), a reduced risk of stroke (by 31%), and a reduction in need for CABG or coronary angioplasty (by 27%).).

Torsades de pointes

Dx: 1. Rapid polymorphic VT. It is a dangerous arrhythmia that often can lead to VFib. 2. It is associated with many factors that prolong the QT interval (e.g., congenital QT syndromes, tricyclic antidepressants, anticholinergics, electrolyte abnormalities, ischemia). Tx: 1. IV magnesium provides cardiac stabilization. 2. Address the underlying cause.

Chest pain

Dx: 1. Rule out any life-threatening causes. These include acute MI, unstable angina, aortic dissection, pulmonary embolus, tension pneumothorax, and esophageal rupture. 2. Assess vital signs. 3. Develop a focused H&P (LQDMASCOT, previous stress tests, echos, cardiac caths, PCI or CABG) 4. Order ancillary tests a. Obtain ECG in almost all cases b. Cardiac enzymes (CK, CK-MB, troponin) depending on clinical suspicion c. Obtain chest radiograph (CXR) in almost all cases d. Under appropriate clinical setting, work up the patient for pulmonary embolism (PE) Tx: 1. It can be difficult to distinguish between GI causes of chest pain and angina. The decision of whether to initiate a cardiac workup is dependent on a patient's overall risk of CAD and the clinical presentation. If patient is young and without risk factors, treat for GERD and follow up if pain recurs. (Anxiety may be a cause but is difficult to diagnose.) 2. An older patient with risk factors should undergo a cardiac workup. 3. There is no fail-proof algorithm for approaching chest pain. In general, have a greater index of suspicion for ischemic causes in the elderly, diabetic populations, and those with a history of CAD 4. If you suspect a cardiac cause of the pain, sublingual nitroglycerin is appropriate. Also give aspirin if the patient does not have a bleeding disorder. If nitroglycerin relieves the pain, a cardiac cause is likely.

Septic shock (most common cause of death in ICU) Septic/neurogenic shock is associated with severe peripheral vasodilatation (flushing, warm skin). Hypovolemic/cardiogenic shock is associated with peripheral vasoconstriction (cool skin). SIRS Characterized by two or more of the following: • Fever (>38°C) or hypothermia (<36°C) • Hyperventilation (rate >20 bpm) or PaCO2 < 32 mmHg • Tachycardia (>90 bpm) • Increased WBC count (>12,000 cell/hpf, <4,000 cells/hpf, or >10% band forms) Sepsis • When blood cultures are positive and SIRS is present • Blood cultures: Obtain two sets from two different sites (each set should have aerobic and anaerobic bottles). Draw blood before antibiotic administration. Septic Shock • Hypotension induced by sepsis persisting despite adequate fluid resuscitation Multiple Organ Dysfunction Syndrome (MODS) • Altered organ function in an acutely ill patient, usually leading to death (kidneys, liver, CNS, etc.)

Dx: 1. Septic shock is essentially a clinical diagnosis. 2. Confirmed by positive blood cultures, but negative cultures are common. 3. A source of infection can aid in diagnosis, but there may be no confirmed source in some cases. Common causes include (but are not limited to) pneumonia, pyelonephritis, meningitis, abscess formation, cholangitis, cellulitis, and peritonitis. Tx: 1. Initially, IV antibiotics (broad spectrum) at maximum dosages. Antibiotics for more rare organisms or antifungal medications may be required if there is no clinical response or if suspicion for an atypical organism (i.e., immunocompromised). If cultures are positive, antibiotics can be narrowed based on sensitivity testing. 2. Surgical drainage if necessary 3. Fluid administration to increase mean BP 4. Vasopressors may be used if hypotension persists despite aggressive IV fluid resuscitation. a. Dopamine is typically the initial agent. b. If dopamine does not increase the BP, norepinephrine may be given.

Premature ventricular complexes PVCs • Couplet: Two successive PVCs • Bigeminy: Sinus beat followed by a PVC • Trigeminy: Two sinus beats followed by a PVC

Dx: 1. Since conduction is not through normal conduction pathways, but rather through ventricular muscle, it is slower than normal, causing a wide QRS. 2. Wide, bizarre QRS complexes followed by a compensatory pause are seen; a P wave is not usually seen because it is "buried" within wide QRS complex. Tx: 1. Most patients are asymptomatic. Some patients may have palpitations and dizziness related to PVCs. If symptomatic, β-blockers may be used. 2. If a patient is found to have frequent PVCs, work-up for underlying structural heart disease should be initiated which may require specific treatment. 3. Patients with frequent, repetitive PVCs and underlying heart disease are at increased risk for sudden death due to cardiac arrhythmia (especially VFib). Order an electrophysiologic study because patients may benefit from an ICD.

Sick sinus syndrome

Dx: 1. Sinus node dysfunction characterized by a persistent spontaneous sinus bradycardia 2. Symptoms include dizziness, confusion, syncope, fatigue, and CHF. Tx: 1. Pacemaker implantation may be required 2. There are three types: • Permanent implantable system for long-term treatment • Temporary systems—either transcutaneous (with electrode pads over chest) or transvenous—both use an external pulse generator that patient can secure to waist with straps 3. On ECG, cardiac pacing is noted by presence of a "spike." 4. Indications • Sinus node dysfunction is most common indication • Symptomatic heart block—Mobitz II second-degree block and complete heart block (even if asymptomatic) • Symptomatic bradyarrhythmias • Tachyarrhythmias to interrupt rapid rhythm disturbances

Sinus bradycardia

Dx: 1. Sinus rate <60 bpm: Clinically significant when rate is persistently <45 bpm Tx: 1. Atropine can elevate the sinus rate by blocking vagal stimulation to the sinoatrial node. 2. A cardiac pacemaker may be required if bradycardia persists.

Nonbacterial verrucous (Libman-Sacks) endocarditis

Dx: 1. Small warty vegetations on both sides of valve leaflets and may present with regurgitant murmurs (associated with systemic lupus erythematosus) 2. Rarely gives rise to infective endocarditis, but can be a source of systematic embolization Tx: 1. Treat underlying SLE 2. Anticoagulate with heparin

Nonbacterial thrombotic (Marantic) endocarditis

Dx: 1. Sterile deposits of fibrin and platelets form along the closure line of cardiac valve leaflets (20% associated with metastatic cancer) 2. Vegetations can embolize to the brain or periphery. Tx: 1. Although the use of heparin may be appropriate, no studies have confirmed its efficacy

Chronic venous insufficiency

Dx: 1. Swelling of the lower leg a. When chronic, causes an aching or tightness feeling of the involved leg; often worse at the end of the day b. Symptoms are worsened by periods of sitting or inactive standing. c. Leg elevation provides relief of symptoms (the opposite is true in arterial insufficiency). 2. Chronic changes include: a. Skin changes (stasis dermatitis) • Skin becomes thin, atrophic, shiny, and cyanotic. • Brawny induration develops with chronicity. b. Venous ulcers • Less painful than ulcers associated with arterial insufficiency • Usually located just above the medial malleolus • Often rapidly recur Tx: 1. Before the development of ulcers, strict adherence to the following controls stasis sequelae in most patients. a. Leg elevation: Periods of leg elevation during the day and throughout the night to a level above the heart. b. Avoiding long periods of sitting or standing c. Heavy-weight elastic stockings (knee-length) are worn during waking hours. 2. If ulcers develop, management also entails: a. Wet-to-dry saline dressings (three times daily) b. Unna venous boot (external compression stocking)—best changed every week to 10 days • Healing occurs in 80% of ulcers. Compliance reduces the rate of recurrence. • For ulcers that do not heal with the Unna boot: Apply split-thickness skin grafts with or without ligation of adjacent perforator veins.

Atrial septal defect Clinical features 1. Mild systolic ejection murmur at pulmonary area secondary to increased pulmonary blood flow 2. Wide, fixed splitting of S2 3. Diastolic flow "rumble" murmur across tricuspid valve area secondary to increased blood flow 4. In advanced disease, signs of RVF may be seen.

Dx: 1. TEE is diagnostic (better than transthoracic echocardiogram). Contrast echocardiography can improve resolution. A so-called "bubble study" is a type of contrast echo which involves injecting microbubbles and watching them cross the defect via a right-to-left shunt. This is often used to aid in diagnosis of ASDs. 2. CXR: Large pulmonary arteries; inc pulmonary markings 3. ECG: Right bundle branch block and right axis deviation; atrial fibrillation/flutter also present Tx: 1. Unless they are very large, most defects do not require closure. 2. Surgical repair when pulmonary-to-systemic blood flow ratio is greater than 1.5:1 or 2:1 or if patient is symptomatic.

Basics of shock A focused history and physical examination to determine possible cause of shock a. Fever/possible site infection suggest septic shock. b. Trauma, GI bleeding, vomiting, or diarrhea suggests hypovolemic shock. c. History of MI, angina, or heart disease suggests cardiogenic shock. If JVD is present, cardiogenic shock. d. If spinal cord injury or neurologic deficits are present, neurogenic shock likely. Shock: Cardiac Output; SVR; PCWP Cardiogenic ↓ ↑ ↑ Hypovolemic ↓ ↑ ↓ Neurogenic ↓ ↓ ↓ Septic ↑ ↓ ↓

Dx: 1. The following signs and symptoms are common to all forms of shock: • Hypotension/tachycardia • Oliguria/lactic acidosis • Altered mental status 2. Simultaneously stabilize patient hemodynamically and determine the cause of shock. a. Establish two large-bore venous catheters, a central line, and an arterial line. b. A fluid bolus (500 to 1,000 mL of normal saline or lactated Ringer's solution) should be given most cases. c. Draw blood: CBC, electrolytes, renal function, PT/PTT d. ECG, CXR e. Continuous pulse oximetry f. Vasopressors (dopamine or norepinephrine) may be given if the patient remains hypotensive despite fluids. g. If the diagnosis is still in question after the above tests, a pulmonary artery catheter (Swan-Ganz catheter) and/or echocardiogram may help diagnosis. Tx: 1. ABCs (airway, breathing, and circulation) should be addressed for all patients in shock. 2. Specific treatment is described below for each type of shock. With the exception of cardiogenic (and sometimes neurogenic) shock, a generous amount of IV fluid is usually required to resuscitate the patient. 3. The more advanced the stage of shock, the greater the fluid (and blood) requirement.

Cholesterol emoblization syndrome

Dx: 1. This syndrome is due to "showers" of cholesterol crystals originating from a proximal source (e.g., atherosclerotic plaque), most commonly the abdominal aorta, iliacs, and femoral arteries. 2. It is often triggered by a surgical or radiographic intervention (e.g., arteriogram), or by thrombolytics. Tx: 1. Treatment is supportive. Do not anticoagulate. Control BP. 2. Amputation or surgical resection is only needed in extreme cases.

Abdominal aortic anuerysm

Dx: 1. Ultrasound a. Test of choice to evaluate both the location and size of the aneurysm b. 100% sensitive in detecting AAAs 2. CT scan a. 100% sensitive in detecting AAAs b. Takes longer to perform than plain radiographs or ultrasound; should only be used in hemodynamically stable patients c. CT is the scan of choice for preoperative planning 3. Abdominal radiographs a. Not routinely used for surveillance of AAAs. Occasionally, an AAA will initially be diagnosed via an abdominal x-ray. b. May show calcifications of the dilated segment (this allows measurement of aortic diameter) c. A quick method of diagnosing AAAs, but a negative study is not helpful, that is, plain films cannot be used to rule out AAAs Tx: 1. Unruptured aneurysms a. Management largely depends on size of aneurysm • If the aneurysm is >5 cm in diameter or symptomatic, surgical resection with synthetic graft placement is recommended. (The infrarenal aorta is replaced with a fabric tube.) This can often be done endovascularly by accessing the femoral artery. The diameter of the normal adult infrarenal aorta is about 2 cm. • The management of asymptomatic aneurysms <5 cm is controversial. Periodic imaging is recommended to follow up growth. No "safe" size exists, however, and small AAAs can rupture. b. Other factors to consider are the patient's life expectancy (patient may be more likely to die of other medical illnesses), and the risk of surgery. 2. Ruptured AAAs: Emergency surgical repair is indicated. All of these patients are unstable (triad of abdominal pain, pulsatile mass, and hypotension). a. While open repair remains the gold standard, some ruptured AAAs may be repaired endovascularly as well.

Superficial thrombophlebitis When superficial thrombophlebitis occurs in different locations over a short period of time, think of migratory superficial thrombophlebitis (secondary to occult malignancy, often of the pancreas). Two conditions that should not be confused with superficial thrombophlebitis are cellulitis and lymphangitis. In these conditions, swelling and erythema are more widespread, and there is no palpable, indurated vein.

Dx: 1. Virchow's triad is again implicated (but pathophysiology not entirely clear) 2. In upper extremities, usually occurs at the site of an IV infusion 3. In lower extremities, usually associated with varicose veins (in the greater saphenous system)—secondary to static blood flow in these veins 4. Pain, tenderness, induration, and erythema along the course of the vein (tender cord may be palpated) Tx: 1. No anticoagulation is required—rarely causes PE 2. Localized thrombophlebitis—mild analgesic (aspirin) is all that is required in most cases; continue activity 3. Severe thrombophlebitis (with pain and cellulitis) a. Bed rest, elevation, and hot compresses b. Once symptoms resolve, ambulation with elastic stockings is recommended. c. Antibiotics usually are not necessary unless the process is suppurative, in which case adequate drainage is indicated. d. Septic phlebitis is usually due to infection of an IV cannula; remove the cannula and administer systemic antibiotics.

Neurogenic shock Neurogenic shock results from a failure of the sympathetic nervous system to maintain adequate vascular tone (sympathetic denervation). Causes include spinal cord injury, severe head injury, spinal anesthesia, pharmacologic sympathetic blockade

Dx: 1. Warm, well-perfused skin 2. Urine output low or normal 3. Bradycardia and hypotension (tachycardia can occur) 4. Cardiac output is dec, SVR low, PCWP low to normal Tx: 1. Judicious use of IV fluids as the mainstay of treatment 2. Vasoconstrictors to restore venous tone, cautiously 3. Supine or Trendelenburg position 4. Maintain body temperature.

Paroxysmal supraventricular tachycardia In accordance with the electrical conducting system of the heart (SA node, atria, AV node, bundle of His, bundle branches, Purkinje fibers) tachycardias can be quickly separated into two categories based on the width of the QRS complex • Narrow QRS complexes suggest that the arrhythmia originates at or above the level of the AV node. • Wide QRS complexes suggest that the arrhythmia originates outside of the normal conducting system or there is a supraventricular arrhythmia with coexisting abnormality in the His-Purkinje system.

Dx: 1. a. AV nodal reentrant tachycardia • Two pathways (one fast and the other slow) within the AV node, so the reentrant circuit is within the AV node • Most common cause of supraventricular tachyarrhythmia (SVT) • Initiated or terminated by PACs • ECG: Narrow QRS complexes with no discernible P waves (P waves are buried within the QRS complex). This is because the circuit is short and conduction is rapid, so impulses exit to activate atria and ventricles simultaneously. b. Orthodromic AV reentrant tachycardia • An accessory pathway between the atria and ventricles that conducts retrogradely • Called a "concealed bypass tract," and is a common cause of SVTs • Initiated or terminated by PACs or PVCs • ECG: Narrow QRS complexes with P waves which may or may not be discernible, depending on the rate. This is because the accessory pathway is at some distance from the AV node (reentrant circuit is longer), and there is a difference in the timing of activation of the atria and ventricles. Tx: 1. Maneuvers that stimulate the vagus delay AV conduction and thus block the reentry mechanism: The Valsalva maneuver, carotid sinus massage, breath holding, and head immersion in cold water (or placing an ice bag to the face) 2. Acute treatment a. Pharmacologic therapy • IV adenosine—agent of choice due to short duration of action and effectiveness in terminating SVTs; works by decreasing sinoatrial and AV nodal activity • IV verapamil (calcium channel blocker) and IV esmolol (β-blocker) or digoxin are alternatives in patients with preserved left ventricular function. • DC cardioversion if drugs are not effective or if unstable; almost always successful 3. Prevention a. Pharmacologic therapy • Digoxin is usually the drug of choice. • Verapamil or β-blockers are alternatives. b. Radiofrequency catheter ablation of either the AV node or the accessory tract (depending on which is the accessory pathway) is preferred if episodes are recurrent and symptomatic.

Cardiac tamponade It is the rate of fluid accumulation that is important, not the amount. a. Two hundred milliliters of fluid that develops rapidly (i.e., blood secondary to trauma) can cause cardiac tamponade. b. Two liters of fluid may accumulate slowly before cardiac tamponade occurs. When fluid accumulates slowly, the pericardium has the opportunity to stretch and adapt to the increased volume (i.e., related to a malignancy). Causes a. Penetrating (less commonly blunt) trauma to the thorax, such as gunshot and stab wounds b. Iatrogenic: Central line placement, pacemaker insertion, pericardiocentesis, etc. c. Pericarditis: Idiopathic, neoplastic, or uremic d. Post-MI with free wall rupture

Dx: Beck's triad (cardiac tamponade) = hypotension, JVD, muffled heart sounds 1. Echocardiogram a. Must be performed if suspicion of tamponade exists based on history/examination b. Usually diagnostic; the most sensitive and specific noninvasive test 2. CXR a. Enlargement of cardiac silhouette when >250 mL has accumulated b. Clear lung fields 3. ECG a. Electrical alternans (alternate beat variation in the direction of the ECG waveforms)— due to pendular swinging of the heart within the pericardial space, causing a motion artifact b. Findings are neither 100% sensitive nor specific. ECG should not be used to diagnose tamponade. 4. Cardiac catheterization a. Shows equalization pressures in all heart chambers b. Shows elevated right atrial pressure with loss of the y descent Tx: 1. Nonhemorrhagic tamponade a. If patient is hemodynamically stable • Monitor closely with echocardiogram, CXR, ECG • If patient has known renal failure, dialysis is more helpful than pericardiocentesis. b. If patient is not hemodynamically stable • Pericardiocentesis is indicated. • If no improvement is noted, fluid challenge may improve symptoms. 2. Hemorrhagic tamponade secondary to trauma a. If the bleeding is unlikely to stop on its own, emergent surgery is indicated to repair the injury. b. Pericardiocentesis is only a temporizing measure and is not definitive treatment. Surgery should not be delayed to perform pericardiocentesis.

Congestive heart failure Systolic dysfunction: a. Owing to impaired contractility (i.e., the abnormality is decreased ejection fraction) b. Causes include: • Ischemic heart disease or after a recent MI—infarcted cardiac muscle does not pump blood (decreased ejection fraction). • Hypertension resulting in cardiomyopathy • Valvular heart disease • Myocarditis (postviral) • Alcohol abuse, radiation, hemochromatosis, thyroid disease Diastolic dysfunction: a. Owing to impaired ventricular filling during diastole (either impaired relaxation or increased stiffness of ventricle or both). Diastolic dysfunction is less common than systolic dysfunction. b. Echocardiogram shows impaired relaxation of left ventricle c. Causes include: • HTN leading to myocardial hypertrophy—most common cause of diastolic dysfunction • Valvular diseases such as aortic stenosis, mitral stenosis, and aortic regurgitation • Restrictive cardiomyopathy (e.g., amyloidosis, sarcoidosis, hemochromatosis) Monitoring a patient with CHF: • Weight—unexplained weight gain can be an early sign of worsening CHF • Clinical manifestations (exercise tolerance is key); peripheral edema • Laboratory values (electrolytes, K, BUN, creatinine levels; serum digoxin, if applicable) Standard treatment of CHF includes a loop diuretic, ACE inhibitor, and beta-blocker. Depending on severity and patient factors, other medications such as digoxin, hydralazine/nitrate, spironolactone may be added. Medications that have shown to lower mortality in CHF: • ACE inhibitors and ARBs • Beta-blockers (carvedilol > metroprolol) • Aldosterone antagonists (spironolactone) • Hydrazaline, plus nitrate General Principles in the Treatment of CH F No one simple treatment regimen is suitable for all patients. The following is a general guideline, but the order of therapy may differ among patients and/or with physician preferences. Mild CHF (NYHA Classes I to II ) • Mild restriction of sodium intake (no-added-salt diet of 4 g sodium) and physical activity • Start a loop diuretic if volume overload or pulmonary congestion is present. • Use an ACE inhibitor as a first-line agent. Mild to Moderate CHF (NYHA Classes II to III ) • Start a diuretic (loop diuretic) and an ACE inhibitor • Add a b-blocker if moderate disease (class II or III) is present and the response to standard treatment is suboptimal. Moderate to Severe CHF (NYHA Classes III to IV) • Add digoxin (to loop diuretic and ACE inhibitor) • Note that digoxin may be added at any time for the relief of symptoms in patients with systolic dysfunction. (It does not improve mortality.) • In patients with class IV symptoms who are still symptomatic despite the above, adding spironolactone can be helpful.

Dx: Tests to order for a new patient with CHF: • CXR (pulmonary edema, cardiomegaly, r/o COPD) • ECG and Cardiac enzymes r/o MI • CBC (anemia) • Echocardiogram (estimate EF, r/o pericardial effusion) • BNP is released from the ventricles in response to ventricular volume expansion and pressure overload. • BNP levels >150 pg/mL correlate strongly with the presence of decompensated CHF. BNP may be useful in differentiating between dyspnea caused by CHF/COPD. • N-terminal pro-BNP (NT-proBNP) is a newer assay with similar predictive value as BNP. The normal range for this value depends on the age of the patient, but a NT-proBNP <300 virtually excludes the diagnosis of HF. 1. Chest x-ray (CXR) a. Cardiomegaly b. Kerley B lines are short horizontal lines near periphery of the lung near the costophrenic angles, and indicate pulmonary congestion secondary to dilation of pulmonary lymphatic vessels. c. Prominent interstitial markings d. Pleural effusion 2. Echocardiogram (transthoracic) a. Initial test of choice—should be performed whenever CHF is suspected based on history, examination, or CXR. b. Useful in determining whether systolic or diastolic dysfunction predominates, and determines whether the cause of CHF is due to a pericardial, myocardial, or valvular process. c. Estimates EF (very important): Patients with systolic dysfunction (EF < 40%) should be distinguished from patients with preserved left ventricular function (EF > 40%). d. Shows chamber dilation and/or hypertrophy 3. ECG is usually nonspecific but can be useful for detecting chamber enlargement and presence of ischemic heart disease or prior MI. 4. Radionuclide ventriculography using technetium-99m ("nuclear ventriculography") a. RBCs tagged with radioisotope are imaged during exercise/rest. b. Provides precise measurement of left and right ventricular EF (and can assess wall motion abnormalities in ischemic heart disease) c. Can be useful when an echocardiogram is technically suboptimal (e.g., severe pulmonary disease), or when more precise assessment of left ventricular function is desired, but in most cases, this test is not ordered even though it is the most accurate test. 5. Cardiac catheterization can provide valuable quantitative information regarding diastolic and systolic dysfunction, and can clarify the cause of CHF if noninvasive test results are equivocal. Consider coronary angiography to exclude CAD as an underlying cause of CHF. 6. Stress testing a. Identifies ischemia and/or infarction b. Quantitates level of conditioning c. Can differentiate cardiac versus pulmonary etiology of dyspnea d. Assesses dynamic responses of HR, heart rhythm, and BP Tx: 1. Systolic dysfunction a. General lifestyle modification: • Sodium restriction (less than 4 g/day) • Weight loss • Smoking cessation • Restrict alcohol use • Exercise program • All patients should monitor weight daily to detect fluid accumulation. • Annual influenza vaccine and pneumococcal vaccine recommended b. Diuretics • Most effective means of providing symptomatic relief to patients with moderate to severe CHF • Recommended for patients with systolic failure and volume overload • Have not been shown to reduce mortality or improve prognosis, just for symptom control. Goal is relief of signs and symptoms of volume overload (dyspnea, peripheral edema) • Loop diuretics: Furosemide (Lasix)—most potent • Thiazide diuretics: Hydrochlorothiazide—modest potency c. Spironolactone (aldosterone antagonist) • Prolong survival in selected patients with CHF • Monitor serum potassium and renal function • Spironolactone is proven effective only for more advanced stages of CHF (classes III and IV). • Eplerenone is an alternative to spironolactone (does not cause gynecomastia). d. ACE inhibitors • Cause venous and arterial dilation, decreasing preload and afterload. • Indicated for left ventricular systolic dysfunction (LV ejection fraction less than 40%). • The combination of a diuretic and an ACE inhibitor should be the initial treatment in most symptomatic patients. • ACE inhibitors reduce mortality (Cooperative North Scandinavian Enalapril Survival Study [CONSENSUS] and Studies of Left Ventricular Dysfunction [SOLVD] trials), prolong survival, and alleviate symptoms in mild, moderate, and severe CHF. • All patients with systolic dysfunction should be on an ACE inhibitor even if they are asymptomatic. • Always start at a low dose to prevent hypotension. • Monitor BP, potassium, BUN, and creatinine. e. Angiotensin II receptor blockers (ARBs) • Used in patients unable to take ACE inhibitors due to side effect of cough, but should not replace ACE inhibitors if patient tolerates an ACE inhibitor f. β-blockers • Proven to decrease mortality in patients with post-MI heart failure. • Reported to improve symptoms of CHF; may slow progression of heart failure by slowing down tissue remodeling. The decrease in heart rate leads to decreased oxygen consumption. Beta-blockers also have antiarrhythmic and antiischemic effect. • Should be given to stable patients with mild to moderate CHF (class I, II, and III) unless there is a noncardiac contraindication • Not all beta-blockers are equal. There is evidence only for metoprolol, bisoprolol, and carvedilol. g. Digitalis • Positive inotropic agent • Useful in patients with EF < 40%, severe CHF, or severe atrial fibrillation (AFib) • Provides short-term symptomatic relief (used to control dyspnea and will decrease frequency of hospitalizations) but has not been shown to improve mortality. • For patients with EF < 40%, who continue to have symptoms despite optimal therapy (with ACE inhibitor, beta-blocker, aldosterone antagonist, and a diuretic). • Serum digoxin level should be checked periodically. h. Hydralazine and isosorbide dinitrates • Can be used in patients who cannot tolerate ACE inhibitors • The combination of hydralazine and isosorbide dinitrate has been shown to improve mortality in selected patients with CHF. But not as effective as ACE inhibitors and require inconvenient dosing schedules. i. The following medications are contraindicated in patients with CHF: • Metformin—may cause potentially lethal lactic acidosis • Thiazolidinediones—causes fluid retention • NSAIDs may increase risk of CHF exacerbation • Some antiarrhythmic agents that have negative ionotropic effects j. The following devices have been shown to reduce mortality in select patients. • An ICD lowers mortality by helping prevent sudden cardiac death (which is the most common cause of death in CHF). It is indicated for patients at least 40 days post-MI, EF < 35%, and class II or III symptoms despite optimal medical treatment. • Cardiac resynchronization therapy (CRT): This is biventricular pacemaker— indications are similar to ICD except these patients also have prolonged QRS duration >120 msec. Most patients who meet criteria for CRT are also candidates for ICD and receive a combined device. k. Cardiac transplantation is the last alternative if the above do not control symptoms. 2. Diastolic dysfunction: Few therapeutic options available; patients are treated symptomatically a. Beta-blockers have clear benefit and should be used b. Diuretics are used for symptom control (volume overload) c. Digoxin and spironolactone should NOT be used. d. ACE inhibitors and ARBs—benefit is not clear for diastolic dysfunction 3. General principles in treatment of CHF (see Clinical Pearls 1-10 and 1-11)


Ensembles d'études connexes

Call of the wild - there is more

View Set

Mendelian Genetics Lab Questions

View Set

Small Business Management Smart Book 4

View Set

Attention Deficit Hyperactivity Disorder (ADHD)

View Set

monster by walter dean myers characters

View Set

Tableau Certified Data Analyst Udemy Course

View Set